Resolución de Problemas en Farmacología Cardiovascular

Escribe (por invitación) Paula Bronn - Ayudante de Farmacología UAI


1-Un paciente de sexo masculino de 51 años de edad ingresa a la guardia tras ser recogido por la ambulancia en la vía pública luego de un episodio sincopal. El médico cardiólogo constata rápidamente taquicardia ventricular de 180 latidos por minuto, y decide administrar un goteo de amiodarona. ¿Considera correcta la administración de amiodarona como primera elección? De no ser así ¿ por qué fármaco optaría? Justifique su decisión.


2-Una paciente de 65 años de edad concurre su consultorio refiriendo palpitaciones. La paciente presenta antecedentes de fibrilación auricular crónica. Al realizarle un ecocardiograma se observa dilatación auricula r izquierda, con una imagen compatible con un trombo intracavitario. Indique cuál de las siguientes le parece la medida terapéutica más apropiada para esta paciente:
a)cardioversión eléctrica de urgencia.
b)Amiodarona y anticoagulación
c)Digitálicos y anticoagulación
d)Beta bloqueantes y bloqueantes cálcicos


3-Un paciente ingresa por guardia presentando disnea de reposo. Al exámen físico se constata: FC 100 por minuto; TA 170/120; rales húmedos en ambas bases y vértices pulmonares; ingurgitación yugular; cianosis periférica.
Ud. decide entonces colocarle una mascara de O2, vasodilatadores, diuréticos e inotrópicos.Elija entre las siguientes la que considere como mejor opción
a) tiazidas – dobutamina.
b) furosemida – dopamina.
c) furosemida – amrinona.
d) furosemida – dobutamina.
Justifique su elección.


4-Una anciana ingresa al servicio de urgencia del hospital presentando: vómitos, diarrea y malestar generalizado. Su hija relata que la paciente se encuentra en tratamiento con Digoxina y diuréticos por insuficiencia cardíaca crónica, a lo que se agrega que durante la última semana ha estado recibiendo además eritromicina (macrólido) por un cuadro neumónico.
Al realizarle un ECG se observa infradesnivel de segmento ST, con una imagen compatible con cubeta digitálica.
¿Cómo explica la intoxicación en esta paciente?
¿Considera Ud que se encuentra ante una urgencia? Justifique.
¿Cuál seria el tratamiento adecuado a instaurar?


5- Ud. recibe un paciente en la guardia al cual le diagnostica angor inestable ¿qué tratamiento farmacológico implementaría en primera instancia?


6- Ud. recibe en su consultorio a un paciente diabético al cual está tratando por hipertensión leve con dieta hiposódica y ejercicio. Al revisar las cifras tensionales tomadas en forma ambulatoria observa que estas aumentaron manteniéndose en casi todas las tomas en cifras cercanas a 160-90 mmg, valores que se repiten en el consultorio.
¿Qué decisión toma con respecto al tratamiento? ¿con qué opciones farmacológicas cuenta?.¿Cual es su opción? Justifique


7- Ud. Recibe en la guardia a un paciente que consulta por hemiparesia izquierda. Al interrogarlo descubre que además presenta disartria.
Ud. decide evaluar la TA y observa que esta se expresa en cifras de 230/160 mmHg.
¿Cómo catalogaría esta situación clínica?.
¿Cuál es la conducta a adoptar con este paciente?
¿Cuál seria el tratamiento farmacológico de elección?


8- Un paciente de 73 años de edad, hipertenso crónico, concurre a su consultorio por presentar: tos seca, irrritativa, de aparición brusca y que refiere como intolerable. Al interrogarlo descubre que hace poco le prescribieron un medicamento cuyo nombre el paciente no recuerda, en reemplazo de los antiguos
antihipertensivos que tomaba.
¿ Que fármaco está recibiendo el paciente? ¿Por qué droga lo reemplazaría teniendo en cuenta la edad del paciente?


9- Una paciente de 59 años de edad,obesa, hipertensa leve, lo consulta por presentar: edema en miembros inferiores, de 2 meses de evolución.
Usted le indica: dieta hipocalórica, hiposódica, ejercicio y un diurético.
¿ Le parece la conducta terapéutica más apropiada para tratar a esta paciente?.¿ Que alternativas a la misma propone?.Justifique.


10- El paciente F.K. ingresa al Servicio de Urgencias presentando: tinitus, cefalea, desorientación temporoespacial, visión borrosa, hipotension y taquicardia.
Sus familiares le informan que se encuentra en tratamiento farmacológico por padecer una arritmia.
Al realizarle el ECG observa prolongación del QT, que comparado con ECG anteriores es de casi un 50% del basal.
¿ Qué fármaco sospecha UD que está recibiendo el paciente?
¿ A que grupo de antiarrítmicos pertenece?.
¿.Cuál es el mecanismo involucrado en la aparición de los síntomas?
¿ Cúal considera que es la conducta terapéutica adecuada a seguir con este paciente? Justifique

Setiembre 2000